NUR 102 Exam 4

अब Quizwiz के साथ अपने होमवर्क और परीक्षाओं को एस करें!

A client is on the surgical unit after orthopedic surgery. The health care provider has prescribed 4 mg of morphine sulfate IM for pain. The vial reads "morphine sulfate 10 mg per 1 mL." How many milliliters should the nurse administer? Record your answer using one decimal place.

0.4 mL

A primary care provider prescribes morphine 4 mg IM for a client in pain. Morphine is supplied as 10 mg/mL. The nurse will administer __________ml. Record your answer using one decimal place.

0.4 mL

Treatment for a child with sinus bradycardia includes atropine 0.02 mg/kg. If the child weighs 20 kg, how much is given per dose? A. 0.02 mg B. 0.04 mg 0.2 mg 0.4 mg

0.4 mg

A cardiologist prescribes digoxin 125 ug ( 0.125 mg ) by mouth every morning for a client diagnosed with heart failure. The pharmacy dispenses tablets that contain 0.25 mg each. How many tablet(s) should the nurse administer in each dose? Record your answer using one decimal place.

0.5 tablets

A nurse is caring for a client with deep vein thrombosis who is scheduled to receive an injection of enoxaparin 75 mg subcutaneously daily. On hand is enoxaparin 100 mg per milliliter (ml). How many milliliter(s) should the nurse administer to the client? Record your answer using two decimal places.

0.75 mL

A pediatric nurse is caring for a 12-year-old client with gonorrhea who is to receive penicillin G 0.6 million units IM as an initial dose. The pharmacy supplies the medication, which is labeled 2.4 million units/4 mL. How many milliliters would the nurse safely administer to this client? Record your answer as a whole number.

1 mL

A 2-year-old child is diagnosed with epiglottitis. Ampicillin is ordered 50 mg/kg/day in six divided doses. The client weighs 12 kg. How many milligrams of ampicillin is given per dose? Record your answer using a whole number.

100 mg

A 10-year-old child is admitted with asthma. The health care provider orders an aminophylline infusion. A loading dose of 6 mg/kg is ordered. The client weighs 30 kg. How many milligrams of aminophylline is contained in the loading dose? Record your answer using a whole number.

180 mg

A client is receiving IV lactated Ringer's solution at a rate of 75 mL/hour for fluid volume replacement. The infusion set provides 15 gtts/mL. At which flow rate should the nurse set the IV? Record your answer using a whole number.

19 gtts/min

A client weighing 167 lb (76 kg) is brought to the emergency department in status epilepticus. The primary care provider asks the nurse to prepare diazepam 0.25 mg/kg. How many milligrams will be given to this client? Round your answer to a whole number.

19 mg

After laparoscopic cholecystectomy, a client reports pain and nausea. A nurse is preparing 75 mg of meperidine and 12.5 mg of promethazine to be administered IM in the same syringe. If the label on the meperidine reads 50 mg/mL and the label on the promethazine reads 25 mg/mL, how many milliliters should the nurse have in the syringe after the correct doses are drawn up? Record your answer using a whole number.

2 mL

A client is ordered to receive 1 g of neomycin sulfate orally every hour × 4 doses followed by 1 g orally every 4 hours for the remaining balance of the 24 days. Neomycin sulfate tablets are available in 500 mg per tablet. How many tablets should the nurse administer for each dose? Record your answer using a whole number.

2 tablets

The health care provider's order reads 2 g of cephalexin PO daily in equally divided doses of 500 mg each. How many times per day should the nurse administer this medication? Record your answer using a whole number.

2 times per day

An infant who weighs 8 kg is to receive ampicillin 25 mg/kg IV every 6 hours. How many milligrams should the nurse administer per dose? Record your answer using a whole number.

200 mg

The nurse is caring for a 12-month-old child with otitis media. The child weighs 11 kg and has no known drug allergies. The primary health care provider has prescribed amoxicillin 200 mg PO every 12 hours. The drug available is amoxicillin suspension 250 mg/5mL. What should the nurse administer per dose? Record your answer using a whole number.

4 mL

The health care provider prescribed t-PA, a thrombolytic agent. The order is for 0.9 mg/kg over 1 hour. The client weighs 110 lb (50 kg). What is the total dose in milligrams the client will receive? Record your answer using a whole number.

45 mg

A 10-year-old client with asthma is prescribed 2 mg of albuterol syrup four times per day. The syrup comes in a dosage strength of 2 mg/5 ml. How many milliliters of syrup should the nurse administer? Record your answer using a whole number.

5 ml

After undergoing small-bowel resection, a client is prescribed metronidazole 500 mg I.V. The mixed I.V. solution contains 100 mL. The nurse is to administer the drug over 30 minutes. The drop factor of the available I.V. tubing is 15 gtt/mL. What is the drip rate in drops per minute? Record your answer using a whole number.

50 drops/minute

Theophylline is ordered for a 1-year-old infant with bronchopulmonary dysplasia. The recommended dosage is 24 mg/kg/day. The child weighs 10 kg. How many milligrams should be given per dose when administered 4 times per day? Record your answer using a whole number.

60 mg

A 3-year-old is to receive 500 mL of dextrose 5% in normal saline solution over 8 hours. At what rate (in milliliters per hour) should a nurse set the infusion pump? Round your answer to a whole number.

63 mL

A health care provider prescribes 150 mg of ibuprofen for a toddler whose temperature didn't lower after receiving acetaminophen. The oral suspension available contains 100 mg per 5 ml. How many milliliters of suspension should the nurse administer? Record your answer using one decimal place.

7.5 mL

A child is to receive furosemide 4 mg/kg/day in one daily dose. The child weighs 20 kg. How many milligrams should the nurse administer in each dose? Record your answer using a whole number.

80 mg

An 83-year-old client has recently lost a spouse of 60 years. What assessment questions demonstrate the nurse's understanding of stressors that are likely to increase the client's risk of depression and limit independence? Select all that apply. A. "Do you consider yourself to be physical healthy?" B. "Are you affiliated with a specific church, synagogue, temple or mosque?" C. "Are you concerned about your finances and being able to pay your bills?" D. "Do you have family living nearby?" E. "Do you see yourself marrying again?"

A. "Do you consider yourself to be physical healthy?" C. "Are you concerned about your finances and being able to pay your bills?" D. "Do you have family living nearby?"

A client presents to the medical clinic for evaluation of a rash and throat tightness. The client reports that the symptoms developed 24 hours after eating peanuts. Which question should the nurse ask the client? A. "Do you have injectable epinephrine available to carry with you at all times?" B. "Have you ever used diphenhydramine to reduce the swelling in your throat?" C. "Does using an albuterol inhaler help you to ease the tightness in your throat?" D. "Are you prescribed prednisone to manage your allergies and airway swelling?"

A. "Do you have injectable epinephrine available to carry with you at all times?"

A client develops hepatic encephalopathy 1 week after portacaval shunt surgery. Her physician prescribes neomycin, 4 g by mouth daily in four divided doses. Her husband asks how neomycin decreases his wife's serum ammonia concentration. How should the nurse respond? A. "It decreases the number of ammonia-producing bacteria in the GI tract." B. "It acidifies the colon and traps ammonia in the GI tract." C. "It binds with ammonia in the GI tract." D. "It increases the growth of such bacteria as Escherichia coli."

A. "It decreases the number of ammonia-producing bacteria in the GI tract."

The nurse is reinforcing education about antihypertensive therapy with the parents of a child with glomerulonephritis. Which statement made by the parent indicates that further teaching is required? A. "My child will need to take antihypertensive drugs for the rest of his life." B. "I should be sure to keep my child's regular appointments." C. "I should watch my child for dizziness and lightheadedness." D. "I will administer the medication at the same time each day."

A. "My child will need to take antihypertensive drugs for the rest of his life."

Parents ask the nurse how they will know when their daughter has reached puberty. What statement indicates that the nurse is providing accurate information to the parents? Select all that apply. A. "She will have irregular menstrual periods." B. "Hip and breast development will become obvious." C. "Increased glandular activity causes an increase in sweat." D. "Her permanent teeth will come in." E. "You will notice that she begins developing pubic hair."

A. "She will have irregular menstrual periods." B. "Hip and breast development will become obvious." E. "You will notice that she begins developing pubic hair."

A client is prescribed haloperidol. When reinforcing the teaching plan about the drug, which instruction would the nurse emphasize? A. "You should report feelings of restlessness or agitation at once." B. "You can take your herbal supplements safely with this drug." C. "Be aware that you'll feel increased energy taking this drug." D. "This drug will indirectly control essential hypertension."

A. "You should report feelings of restlessness or agitation at once."

Milk of magnesia does not relieve a client's constipation. The physician orders a soap suds enema, 500 mL. How many liters will the nurse administer? A. 0.5 L B. 0.75 L C. 1 L D. 2 L

A. 0.5 L

A client is admitted for an amniocentesis. Initial data collection findings include the following: 16 weeks pregnant, vital signs within normal limits, hemoglobin 12.2 gm, hematocrit 35%, and type O-negative blood. Which action would be most important to include in the client's plan of care after the amniocentesis has been completed? A. Administer RhoGAM. B. Check for rupture of membranes. C. Assess uterine activity. D. Provide additional fluid.

A. Administer RhoGAM.

A client who sustained a head injury in a motor vehicle accident is prescribed phenytoin liquid to prevent seizures. The client is unable to take anything by mouth and has a feeding tube in place for enteral feedings. Which intervention by the nurse is most appropriate when administering phenytoin to this client? A. Administering the phenytoin 2 hours before or 2 hours after beginning the tube feedings B. Asking the registered nurse to administer the phenytoin I.V. C. Monitoring the phenytoin level closely because enteral feedings increase the drug level D. Assessing for signs of bleeding

A. Administering the phenytoin 2 hours before or 2 hours after beginning the tube feedings

A home health nurse visits a client who's taking pilocarpine, a miotic agent, to treat glaucoma. The nurse notes that the client's pilocarpine solution is cloudy. What should the nurse do first? A. Advise the client to discard the drug because it may have undergone chemical changes or become contaminated. B. Advise the client to obtain a fresh container of pilocarpine solution to avoid omitting prescribed doses. C. Observe the client or a family member administer the drug to determine possible contamination sources. D. Advise the client to keep the container closed tightly and protected from light.

A. Advise the client to discard the drug because it may have undergone chemical changes or become contaminated.

What should the nurse do when administering pilocarpine? A. Apply pressure on the inner canthus to prevent systemic absorption. B. Administer at bedtime to prevent night blindness. C. Apply pressure on the outer canthus to prevent adverse reactions. D. Flush the client's eye with normal saline solution to prevent burning.

A. Apply pressure on the inner canthus to prevent systemic absorption.

The licensed practical nurse discovers a client with a pulse rate of 40 beats/minute. His blood pressure is 80/50 mm Hg and he reports dizziness. Which medication would the registered nurse use to treat the client's bradycardia? A. Atropine B. Dobutamine C. Amiodarone D. Lidocaine

A. Atropine

A nurse is monitoring a client receiving intravenous (IV) fluid via pump. The alarm of the pump starts to beep for occlusion. What should the nurse do first? A. Check the roller clamp. B. Shut off the pump. C. Increase the rate of infusion to flush the line. D. Flush the IV line with heparin solution.

A. Check the roller clamp.

After being treated with heparin for a pulmonary embolism, a client is prescribed warfarin using a sliding scale. Which action should the nurse take before administering this drug? A. Closely monitor prothrombin time (PT) and international normalized ratio (INR) results to determine the dose of warfarin to administer. B. Notify the physician of PT and INR results before administering the next dose of warfarin. C. Administer the next dose of warfarin and then notify the physician if PT and INR results are abnormally high. D. Administer the next dose of warfarin and then notify the physician if PT and INR results are abnormally low.

A. Closely monitor prothrombin time (PT) and international normalized ratio (INR) results to determine the dose of warfarin to administer.

During labor, a client's cervix fails to dilate progressively, despite her uncomfortable uterine contractions. To augment labor, the physician orders oxytocin. When preparing the client for oxytocin administration, the nurse describes the contractions the client is likely to feel when she starts to receive the drug. Which description is accurate? A. Contractions will be stronger and more uncomfortable and will peak more abruptly. B. Contractions will be weaker, longer, and more effective. C. Contractions will be stronger, shorter, and less uncomfortable. D. Contractions will be stronger and shorter and will peak more slowly.

A. Contractions will be stronger and more uncomfortable and will peak more abruptly.

Which potential physical changes should the nurse discuss with the older adult client? Select all that apply. A. Decrease in visual acuity B. Decrease in joint mobility C. Decrease in fluid needs D. Decreased sense of balance E. Decrease auditory acuity

A. Decrease in visual acuity B. Decrease in joint mobility D. Decreased sense of balance E. Decrease auditory acuity

A nurse is administering a newly prescribed I.V. antibiotic to a client who suddenly develops wheezing and dyspnea. Which is the nurse's priority action? A. Discontinue the antibiotic infusion. B. Administer diphenhydramine I.V. C. Administer epinephrine I.M. D. Start 100% oxygen using a nonrebreather mask.

A. Discontinue the antibiotic infusion.

A client is admitted to the emergency department with an acute asthma attack. The physician prescribes ephedrine sulfate, 25 mg subcutaneously (subQ). After administration, how soon should the nurse expect ephedrine take effect? A. Immediately B. In 3 minutes C. In 1 hour D. In 2 hours

A. Immediately

For a client with an exacerbation of rheumatoid arthritis, the physician prescribes the corticosteroid prednisone. When caring for this client, the nurse should monitor for which adverse drug reactions? A. Increased weight, hypertension, and insomnia B. Vaginal bleeding, jaundice, and inflammation C. Stupor, breast lumps, and pain D. Dyspnea, numbness, and headache

A. Increased weight, hypertension, and insomnia

A client, age 23, is diagnosed with type 1 diabetes. The physician prescribes 15 units of U-100 regular insulin and 35 units of U-100 isophane insulin suspension (NPH) to be taken before breakfast. The nurse checks the medication order, assembles the appropriate equipment, washes her hands, rotates the NPH insulin vial, puts on disposable gloves, and cleans the stoppers. To draw the two insulin doses into the single U-100 insulin syringe, which sequence should the nurse use? A. Inject 35 units of air into the NPH vial; inject 15 units of air into the regular insulin vial and withdraw 15 units of regular insulin; and withdraw 35 units of NPH. B. Inject 15 units of air into the regular insulin vial; inject 35 units of air into the NPH vial and withdraw 35 units of NPH; and withdraw 15 units of regular insulin. C. Inject 15 units of air into the regular insulin vial and withdraw 15 units of regular insulin; inject 35 units of air into the NPH vial and withdraw 35 units of NPH. D. Inject 35 units of air into the NPH vial; inject 15 units of air into the regular insulin vial; withdraw 35 units of NPH; and withdraw 15 units of regular insulin.

A. Inject 35 units of air into the NPH vial; inject 15 units of air into the regular insulin vial and withdraw 15 units of regular insulin; and withdraw 35 units of NPH.

Which nursing intervention takes highest priority when caring for a client who's receiving a blood transfusion? A. Instructing the client to report any itching, swelling, or dyspnea B. Informing the client that the transfusion usually takes 1½ to 2 hours C. Documenting blood administration in the client care record D. Checking the client's vital signs when the transfusion ends

A. Instructing the client to report any itching, swelling, or dyspnea

Atropine is being administered to a child with sinus bradycardia. Which statement is most accurate about the administration of this medication? A. It increases heart rate. B. It raises blood pressure. C. It dilates bronchial tubes. D. It decreases heart rate.

A. It increases heart rate.

A client with chronic obstructive pulmonary disease (COPD) takes anhydrous theophylline, 200 mg by mouth every 8 hours. During a routine clinic visit, the client asks the nurse how the drug works. What mechanism of action of anhydrous theophylline in treating a nonreversible obstructive airway disease such as COPD will the nurse share with the client? A. It makes the central respiratory center more sensitive to carbon dioxide and stimulates the respiratory drive. B. It inhibits the enzyme phosphodiesterase, which decreases the degradation of cyclic adenosine monophosphate, a bronchodilator. C. It stimulates adenosine receptors, causing bronchodilation. D. It alters diaphragm movement, which increases chest expansion and enhances the lung's capacity for gas exchange.

A. It makes the central respiratory center more sensitive to carbon dioxide and stimulates the respiratory drive.

A client is prescribed metformin to control type 2 diabetes. The nurse should monitor for which life-threatening adverse reaction? A. Lactic acidosis B. Nausea C. Vomiting D. Megaloblastic anemia

A. Lactic acidosis

A client is prescribed digoxin 0.125 mg by mouth stat. The pharmacy dispenses digoxin 0.25 mg. The nurse promptly administers the medication and then realizes the incorrect dose has been administered. How should the nurse proceed? A. Obtain vital signs, and immediately notify the primary health care provider and charge nurse of the error. B. Obtain a copy of the primary health care provider's prescription, and inform the pharmacy of the dispensing error. C. Immediately inform the pharmacist of his dispensing error, and document the incident. D. Inform the pharmacist and the nursing supervisor of the error, and document the incident.

A. Obtain vital signs, and immediately notify the primary health care provider and charge nurse of the error.

A nurse needs to administer prescribed medications to a client with heart failure. Prior to administering the medications, what actions should the nurse take? Select all that apply. A. Perform handwashing. B. Hold all the medications until the primary health care provider has examined the client. C. Check the client's medical record number and name on the identification bracelet. D. Check the client's allergies in the medical record, and verify them with the client. E. Ask the client if there are any medications that will be refused.

A. Perform handwashing. C. Check the client's medical record number and name on the identification bracelet. D. Check the client's allergies in the medical record, and verify them with the client.

A geriatric client has experienced several adverse drug reactions. What does the nurse recognize that this client may benefit from? A. Reduced drug dosages B. Nursing home placement C. Increased drug doses at longer intervals D. Frequent visits to the physician

A. Reduced drug dosages

A nurse is working on the oncology unit when a chemotherapy drug spills on the floor. What should the nurse do next? A. Restrict access to area of the spill. B. Use nonabsorbent material to clean the floor. C. Clean the area with soap and water. D. Dispose of all supplies used in cleaning in a regular garbage bag.

A. Restrict access to area of the spill.

Which religions require mandatory baptism for a dying child? Select all that apply A. Roman Catholic B. Russian Orthodox C. Greek Orthodox D. Quakers E. Hindu

A. Roman Catholic B. Russian Orthodox C. Greek Orthodox

A hospital is conducting a root cause analysis for a serious medication error made by a nurse that injured a client. What is the expected outcome of the root cause analysis? A. The cause of the error is identified through system-wide analysis. B. The nurse is terminated for making such an error. C. The client's family sues the hospital. D. The pharmacist that processed the order is terminated.

A. The cause of the error is identified through system-wide analysis.

The newly hired graduate nurse asks the nurse preceptor what is the only advantage of using a floor stock system. Which rationale does the preceptor give the graduate nurse? A. The nurse can implement medication orders quickly. B. The nurse receives input from the pharmacist. C. The system minimizes transcription errors. D. The system reinforces accurate calculations.

A. The nurse can implement medication orders quickly.

A client is admitted to the orthopedic unit for treatment of a fractured right femur caused by a motor vehicle crash. He is scheduled to undergo an open reduction internal-fixation of his right femur. The night before surgery, the nurse administers 250 mg of glutethimide as prescribed. Which statement regarding usage of glutethimide is correct? A. The nurse should store the drug in a tight, light-resistant container. B. The nurse should dilute it in fruit juice to improve absorption. C. The nurse shouldn't use the liquid if it becomes slightly darkened. D. The nurse should discard unused portions of the drug immediately after use.

A. The nurse should store the drug in a tight, light-resistant container.

The nurse educator is preparing an in-service on administration of sustained-release tablets. Which true statement about sustained-release tablets would the educator include in the preparation? A. They should never be split, crushed, or chewed. B. They should never be split or crushed, but they may be chewed. C. They should never be chewed, but they may be split or crushed. D. They may be split, crushed, or chewed, depending on the client's condition.

A. They should never be split, crushed, or chewed.

Lorazepam is often given along with a neuroleptic agent, such as haloperidol. What is the purpose of administering the drugs together? A. To reduce anxiety and potentiate the sedative action of the neuroleptic B. To counteract extrapyramidal effects of the neuroleptic C. To manage depressed clients D. To increase the client's level of awareness and concentration

A. To reduce anxiety and potentiate the sedative action of the neuroleptic

Which client would be most at risk for secondary Parkinson disease caused by pharmacotherapy? A. a 30-year-old client with schizophrenia taking chlorpromazine B. a 50-year-old client taking nitroglycerin tablets for angina C. a 60-year-old client taking prednisone for chronic obstructive pulmonary disease (COPD) D. a 75-year-old client using naproxen for rheumatoid arthritis

A. a 30-year-old client with schizophrenia taking chlorpromazine

A hospitalized client asks the nurse for "something for pain." Which information is most important for the nurse to gather before administering the medication? Select all that apply. A. administration time of the last dose B. client's pain level on a scale of 1 to 10 C. type of medication the client has been taking D. client's reaction to the previous dose E. client's most current height and weight F. effectiveness of prior dose of medication

A. administration time of the last dose B. client's pain level on a scale of 1 to 10 C. type of medication the client has been taking D. client's reaction to the previous dose F. effectiveness of prior dose of medication

To combat the most common adverse effects of chemotherapy, the nurse would administer an: A. antiemetic. B. antimetabolite. C. antibiotic. D. anticoagulant.

A. antiemetic.

On discharge after treatment for alcoholism, a client plans to take disulfiram as prescribed. When teaching the client about this drug, the nurse emphasizes the need to: A. avoid all products containing alcohol. B. adhere to concomitant vitamin B therapy. C. return for monthly blood drug level monitoring. D. limit alcohol consumption to a moderate level.

A. avoid all products containing alcohol.

A client comes to the clinic with back pain that has been unrelieved by continuous ibuprofen use over the past several days. Current prescription medications include captopril and hydrochlorothiazide. Which laboratory value should the nurse report? A. blood urea nitrogen (BUN) of 26 mg/dL and serum creatinine of 2.35 mg/dL B. sodium of 145 mEq/L and potassium of 5.4 mEq/L C. creatine phosphokinase of 21 U/L D. white blood cell count of 9,000 cells/mm3

A. blood urea nitrogen (BUN) of 26 mg/dL and serum creatinine of 2.35 mg/dL

A client with severe preeclampsia is receiving an intravenous infusion of magnesium sulfate. The client is exhibiting signs and symptoms of magnesium toxicity. Which medication would the nurse expect to be given? A. calcium gluconate B. hydralazine C. naloxone D. Rho(D) immune globulin

A. calcium gluconate

After undergoing a thyroidectomy, a client develops hypocalcemia and tetany. Which medication should the nurse anticipate administering? A. calcium gluconate B. potassium chloride C. sodium bicarbonate D. sodium phosphorus

A. calcium gluconate

A histamine (H2) receptor antagonist is prescribed for a client with recurrent gastrointestinal discomfort. The nurse is instructing the client from a medication pamphlet and highlights which medications in this classification? Select all that apply. A. cimetidine B. ranitidine C. nizatidine D. famotidine E. esomeprazole

A. cimetidine B. ranitidine C. nizatidine D. famotidine

A client is admitted to the hospital displaying sinus bradycardia, nausea, anorexia, and blurred vision. What should the nurse suspect this client to be experiencing? A. digoxin toxicity B. myocardial infarction C. hypertensive crisis D. cor pulmonale

A. digoxin toxicity

A student nurse is reviewing physician orders written on a client's chart. Which entry is written incorrectly because it contains material from the "do not use" list of the Joint Commission on Accreditation of Healthcare Organizations (Joint Commission)? A. epoetin alfa 6500 U SQ daily. B. acetaminophen 550 mg po every 4 hours for fever greater than 102 degrees F. C. diazepam 5 mg po on-call to the OR. D. levothyroxine sodium 0.125 mcg po daily.

A. epoetin alfa 6500 U SQ daily.

At a previous visit, the parents of an infant with cystic fibrosis (CF) received instruction from the nurse in the administration of pancrelipase. At a follow-up visit, which finding in the infant suggests that the parents are not administering the pancreatic enzymes as instructed? A. fatty stools B. bloody urine C. bloody stools D. glucose in urine

A. fatty stools

The nurse is reviewing the content of a prescription before giving it to a client. The nurse determines that the prescription is accurately written when which information is included on the prescription? Select all that apply. A. healthcare provider signature B. frequency C. pharmacy name D. dose E. telephone number of client

A. healthcare provider signature B. frequency D. dose

Which nursing action is most important to decrease the risk of postoperative complications in a child with sickle cell anemia? A. increasing fluids B. preparing the child psychologically C. discouraging coughing D. limiting the use of analgesics

A. increasing fluids

A nurse is administering eyedrops to a client with glaucoma. To achieve maximum absorption, where in the eye should the nurse instill the eyedrops? A. into the conjunctival sac B. on the pupil C. on the sclera D. into the vitreous humor

A. into the conjunctival sac

Which additional health care provider order should a nurse anticipate for a client who has been prescribed corticosteroids? A. perform blood glucose checks every six hours. B. restrict fluids to 1,000 ml in 24 hours. C. administer lactulose 40 g in 4 oz of water daily. D. obtain complete blood count (CBC) every 12 hours.

A. perform blood glucose checks every six hours.

A 1-year-old child is brought by a parent to the clinic with a rash on the abdomen and is diagnosed with scabies. What first line medication for the treatment of scabies does the nurse anticipate reinforcing education about? A. permethrin cream 5% B. crotamiton lotion 10% C. lindane lotion 1% D. ivermectin orally

A. permethrin cream 5%

A nurse is caring for a client who received 1 unit of fresh frozen platelets (FFP) for a platelet count of 20,000 mm3. Which repeat laboratory values will be of greatest concern to the nurse? A. platelet count 22,000 mm3 B. blood urea nitrogen 20 mg/dL C. white blood cell count 4.8 µL D. red blood cell count 5.2 µL

A. platelet count 22,000 mm3

The nurse is obtaining vital signs from a client who is receiving an intravenous antibiotic for the first time. Which observation made by the nurse requires immediate intervention? Select all that apply. A. rash on skin of face, chest, and arms B. reports severe itching all over C. inspiratory wheezes D. heart rate of 86 E. reports mouth is dry

A. rash on skin of face, chest, and arms B. reports severe itching all over C. inspiratory wheezes

Sildenafil has been prescribed for a client. While reviewing his medical records, the nurse would question which order? A. use of nitroglycerin B. insomnia C. use of multivitamins D. neuralgia

A. use of nitroglycerin

An adolescent with diabetes is learning to mix regular insulin and NPH insulin in the same syringe. Which action, if performed by the teen, would indicate the need for further instruction? A. withdraws the NPH insulin first B. injects air into the NPH insulin bottle first C. after drawing up the first insulin, removes air bubbles from the syringe D. injects an amount of air equal to the desired dose of insulin

A. withdraws the NPH insulin first

A client comes to the emergency department with symptoms of a myocardial infarction (MI). The health care provider prescribes reteplase. The nurse is aware that this medication will be most effective when given at which time? A. within 1 to 3 hours of onset of symptoms B. within 6 hours of onset of symptoms C. within 12 hours of onset of symptoms D. within 6 to 8 hours of onset of symptoms

A. within 1 to 3 hours of onset of symptoms

A school-age child experiences symptoms of excessive polyphagia, polyuria, and weight loss. The physician diagnoses type I diabetes mellitus and admits the child to the facility for insulin regulation. The physician prescribes an insulin regimen of insulin (Humulin R) and isophane insulin (Humulin N) administered subcutaneously. How soon after administration can the nurse expect the regular insulin to begin to act? A. ½ to 1 hour B. 1 to 2 hours C. 4 to 8 hours D. 8 to 10 hours

A. ½ to 1 hour

After reinforcing education to a client on how to correctly self-administer daily maintenance dose of 3 units of regular insulin and 4 units of NPH insulin, which client statement demonstrates that the education has been successful? A. "I'll check my blood sugar after breakfast and give myself shots in my stomach." B. "After taking my insulin out of the refrigerator, I'll draw up the clear insulin first to the line for 3 units and then cloudy insulin until there's a total of 7 units in the syringe." C. "First, I'll check my blood sugar; then I'll get the insulin from the refrigerator and withdraw 7 units." D. "I should inject the insulin into a different site each time and then put it back in the pantry for safekeeping."

B. "After taking my insulin out of the refrigerator, I'll draw up the clear insulin first to the line for 3 units and then cloudy insulin until there's a total of 7 units in the syringe."

A client with a urinary tract infection is prescribed co-trimoxazole. The nurse should provide which medication instruction? A. "Take the medication with food." B. "Drink at least eight 8-oz glasses of fluid daily." C. "Avoid taking antacids during co-trimoxazole therapy." D. "Don't be afraid to go out in the sun."

B. "Drink at least eight 8-oz glasses of fluid daily."

A client is given triazolam for a sleep disorder. The nurse is reinforcing some teaching precautions concerning the medication. Which statements by the client indicate an understanding of the information provided? A. "I should take the medication with citrus juice." B. "I shouldn't confuse this medication with Haldol." C. "It's okay to take a short drive after taking the medication." D. "It's okay to smoke while I take this medication."

B. "I shouldn't confuse this medication with Haldol."

A client has been prescribed an anti-inflammatory for osteoarthritis, and the nurse has reinforced educating the client about taking the medication. Which statement by the client indicates that the nurse's education has been effective? A. "If I'm not free from pain in a week, I'll come back to the clinic." B. "It can take up to 2 to 3 weeks for me to feel the full effects from the medication." C. "I'll increase my dose if I'm not better in a few days." D. "If I don't experience pain relief in a few days, I need to stop taking the medication."

B. "It can take up to 2 to 3 weeks for me to feel the full effects from the medication."

A pregnant client asks the nurse whether she can take castor oil for her constipation. How should the nurse respond? A. "Yes, it produces no adverse effects." B. "No, it can initiate premature uterine contractions." C. "No, it can promote sodium retention." D. "No, it can lead to increased absorption of fat-soluble vitamins."

B. "No, it can initiate premature uterine contractions."

An elderly client asks a nurse how to treat chronic constipation. What is the best recommendation the nurse can make? A. "Taking magnesium citrate when necessary will help." B. "Take a stool softener, such as docusate sodium (Colace), daily." C. "Use a tap-water enema weekly to evacuate the rectum." D. "Administer a phospho-soda enema when necessary."

B. "Take a stool softener, such as docusate sodium (Colace), daily."

A client with rheumatoid arthritis reports flatulence and heartburn after taking piroxicam. Which instruction should the nurse reinforce to address the client's concern? A. "These side effects will subside as you continue to take the medication." B. "Take an antacid at the same time that you take the medication." C. "This medication is used for short-term treatment of your arthritis." D. "Try taking a lower dose of the medication to relieve your symptoms."

B. "Take an antacid at the same time that you take the medication."

A client is receiving pilocarpine eye drops. Which statement made by the client shows correct understanding of the medication? A. "The medication will help dilate the pupils of my eyes." B. "The medication will help decrease pressure in my eyes." C. "The medication will prevent eye infection." D. "The medication will prevent eye movement."

B. "The medication will help decrease pressure in my eyes."

A client is prescribed a corticosteroid inhaler along with a bronchodilator inhaler. Which instruction about these drugs should the nurse give the client? A. "Notify your physician if your heart rate increases by more than 50 beats/minute after using these medications." B. "Use the bronchodilator first, then wait about 5 minutes before using the corticosteroid." C. "Use the bronchodilator whenever you feel you need it." D. "You should be able to take almost any over-the-counter medication you feel you need."

B. "Use the bronchodilator first, then wait about 5 minutes before using the corticosteroid."

The nurse is preparing a client who has been newly diagnosed with asthma for discharge. As part of his discharge orders, the client is prescribed albuterol via nebulizer every 8 hours for 3 days, followed by one dose daily thereafter. Which instruction should the nurse include when teaching the client about nebulizer use? A. "You can be flexible with scheduling your albuterol treatments." B. "You should take your pulse before and after treatment; if your pulse rate increases by more than 30 beats/minute you should notify your physician." C. "If you feel short of breath you can use your nebulizer more frequently than prescribed." D. "You might develop nervousness and palpitations during your treatment; this is normal and will subside."

B. "You should take your pulse before and after treatment; if your pulse rate increases by more than 30 beats/minute you should notify your physician."

The nurse is preparing to give a 9-year-old client a preoperative I.M. injection. Which size needle should the nurse use? A. 22G, 1½" B. 22G, 1" C. 20G, 1½" D. 20G, 1"

B. 22G, 1"

The nurse is caring for a client who is receiving warfarin. The nurse reinforces to the client that anticoagulant effects may not be seen for how many days? A. 1-2 B. 3-5 C. 6-8 D. 9-11

B. 3-5

The nurse is caring for an infant who is receiving I.V. therapy. The health care provider orders D5NS 400-mL to infuse in 8 hours. How much I.V. solution would the nurse place in the Buretrol? A. 30-mL B. 50-mL C. 100-mL D. 150-mL

B. 50-mL

The physician prescribes norfloxacin, 400 mg by mouth twice daily, for a client with a urinary tract infection (UTI). The client asks the nurse how long to continue taking the drug. The nurse advises the client to take the medication for how many days? A. 3 to 5 days B. 7 to 10 days C. 12 to 14 days D. 10 to 21 days

B. 7 to 10 days

A client with a diagnosis of tuberculosis (TB) is informed he will have to take medication for the treatment of the disease. How long does the nurse inform the client he will have to be compliant with treatment? A. 2 to 4 months B. 9 to 12 months C. 18 to 24 months D. more than 2 years

B. 9 to 12 months

A nurse is reinforcing education with a client about three medications that the client will receive after discharge. While performing the discharge education, the nurse notices that the client suddenly becomes withdrawn and appears anxious. What action should the nurse take? A. Notify the primary health care provider, and request a change in the prescriptions. B. Acknowledge the client's behavior, and seek clarification. C. Request that the primary health care provider prescribe generic alternatives. D. Explore with the client whether the client can purchase the medications over an extended period.

B. Acknowledge the client's behavior, and seek clarification.

After a client receives an I.M. injection, he complains of burning pain in the injection site. Which nursing action would be the best to take at this time? A. Apply a cold compress to decrease swelling. B. Apply a warm compress to dilate the blood vessels. C. Massage the area to promote absorption of the drug. D. Instruct the client to tighten his gluteal muscles to promote better absorption.

B. Apply a warm compress to dilate the blood vessels.

Which nursing action is appropriate when administering a glycerin suppository to a client? A. Removing the suppository from the refrigerator 30 minutes before insertion B. Applying a lubricant to the suppository C. Assisting the client to a right-side lying position with the left leg flexed upward D. Instructing the client to bear down during insertion

B. Applying a lubricant to the suppository

A client is upset to learn that corticosteroids need to be taken to control symptoms of systemic lupus erythematosus (SLE). While the nurse is preparing to administer medication, the client refuses to take it, stating, "This is turning me into an old woman before my time." What is the best response by the nurse? A. Explain that the symptoms of the disease are chronic and progressive and much worse than the side effects from the drugs. B. Ask about the medication side effects that are a concern and explain why suddenly stopping the drug can cause problems. C. Encourage the client to take the medication until able to consult with her physician regarding the side effects. D. Document the refusal to take the medication and notify the physician.

B. Ask about the medication side effects that are a concern and explain why suddenly stopping the drug can cause problems.

After an instructor has posted assignments, a person claiming to be a nursing student arrives on a unit and asks a nurse for access to the medication records of a client to whom the student nurse has been assigned. The student's only identification (ID) is a laboratory coat with the school's name on it. What is the nurse's most appropriate response? A. Allow the student access to the medication record because the instructor has posted an assignment sheet. B. Ask the student to provide a photo ID for comparison with the names on the assignment sheet. C. Ask the student to contact the instructor by phone to verify the student's identification. D. Allow the student supervised access to the client's medication record.

B. Ask the student to provide a photo ID for comparison with the names on the assignment sheet

The nurse is caring for a client with a history of kidney cysts. The client is having right-sided flank pain and is scheduled to have an intravenous pyelogram with iodine contrast solution. The client states that a test was performed before but the name is unknown to the client. The client also tells the nurse that after "something" was injected into the I.V., the client's throat "started closing." What should the nurse do? Select all that apply. A. Have the client go for the test. B. Call the healthcare provider to discuss the previous reaction. C. Cancel the test. D. Verify the test order with the healthcare provider. E. Have the healthcare provider order another test.

B. Call the healthcare provider to discuss the previous reaction. D. Verify the test order with the healthcare provider.

The nurse is initiating an intravenous (IV) access for a client who needs an infusion of normal saline solution. Which nursing action should the nurse perform before the venipuncture? A. Prime the IV tubing before initiating the intravenous access. B. Check for latex allergy before applying the tourniquet. C. Check laboratory values for electrolytes. D. Use the biggest needle size for infusion.

B. Check for latex allergy before applying the tourniquet.

A physician's order states to administer lorazepam, 20 mg by mouth twice per day, to treat anxiety. How should the nurse proceed? A. Administer the dose after dissolving the tablet in 30 ml of diluent. B. Clarify the order with the prescribing physician because the amount prescribed exceeds the recommended dose. C. Administer the first dose when the client requests it. D. Question the prescribing physician about the use of the drug because it isn't indicated for anxiety.

B. Clarify the order with the prescribing physician because the amount prescribed exceeds the recommended dose.

A client has a nasogastric (NG) tube. The physician prescribes an oral medication that is not available in liquid form. Which action should the nurse utilize to administer the tablet form to this client? A. Dissolve the tablets, and then pour the liquid down the NG tube. B. Crush the tablets and prepare a liquid form, and then insert it into the NG tube using a syringe. C. Cut the tablets in half and wash them down the NG tube, using a syringe filled with water. D. Crush the tablets and wash the powder down the NG tube, using a syringe filled with saline solution.

B. Crush the tablets and prepare a liquid form, and then insert it into the NG tube using a syringe.

Which major physical changes should the nurse identify as being related to the normal aging process? Select all that apply. A. Decreased need for exercise B. Decreased functioning of organs C. Decreased reaction time D. Decreased nutritional requirements E. Decreased capacity for recovery from injury or illness

B. Decreased functioning of organs C. Decreased reaction time E. Decreased capacity for recovery from injury or illness

A client diagnosed with pneumonia refuses the prescribed oral antibiotic. The client is alert and oriented, vital signs are within normal range, and crackles are scattered throughout the posterior left lower lobe of the client's lung. Which actions should the nurse perform? Select all that apply. A. Mix the medication into the client's food without the client's knowledge. B. Document that the client refused to take the medication. C. Address the client's concern about the medication by clarifying its purpose. D. Leave the medication with the client in case the client decides to take it at a later time. E. Notify the physician.

B. Document that the client refused to take the medication. C. Address the client's concern about the medication by clarifying its purpose. E. Notify the physician.

A nurse is reinforcing education to a client with prostatitis who is receiving co-trimoxazole double strength. Which education is appropriate for this client? A. Do not expect improvement of symptoms for 7 to 10 days. B. Drink six to eight glasses of fluid daily while taking this medication. C. If a sore mouth or throat develops, take the medication with milk or an antacid. D. Use a sunscreen of at least SPF-15 with PABA to protect against drug-induced photosensitivity.

B. Drink six to eight glasses of fluid daily while taking this medication.

The nurse is monitoring a client receiving tranylcypromine sulfate. Which serious adverse reaction can occur with high dosages of this monoamine oxidase (MAO) inhibitor? A. Hypotensive episodes B. Hypertensive crisis C. Muscle flaccidity D. Hypoglycemia

B. Hypertensive crisis

For a client newly diagnosed with radiation-induced thrombocytopenia, the nurse should include which intervention in the plan of care? A. Administering aspirin if the temperature exceeds 102° F (38.8° C) B. Inspecting the skin for petechiae once every shift C. Providing frequent rest periods D. Placing the client in strict isolation

B. Inspecting the skin for petechiae once every shift

A nurse is preparing a presentation for a pregnant client with diabetes. Which information would the nurse include to explain why a pregnant diabetic client is at risk for having a large-for-gestational-age infant? A. Excess sugar causing reduced placental functioning B. Insulin acting as a growth hormone on the fetus C. Maternal dietary intake of high calories D. Excess insulin reducing placental functioning

B. Insulin acting as a growth hormone on the fetus

A client takes 30 ml of magnesium hydroxide and aluminum hydroxide with simethicone by mouth 1 hour and 3 hours after each meal and at bedtime for treatment of a duodenal ulcer. Why does the client take this antacid so frequently? A. It has a slow onset of action. B. It has a short duration of action. C. It has a prolonged half-life. D. It's highly metabolized.

B. It has a short duration of action.

A nurse is caring for a client who's taking the anticoagulant warfarin (Coumadin). Which instruction regarding warfarin therapy should the nurse give to the client? A. Report incidents of diarrhea. B. Limit foods high in vitamin K. C. Use a straight razor when shaving. D. Take aspirin for pain relief.

B. Limit foods high in vitamin K.

A male client is receiving digoxin and furosemide to treat heart failure. He reports feeling weak and having muscle cramps. His apical pulse is 76 beats/minute; respirations, 16 breaths/minute; and blood pressure, 148/86 mm Hg. What action should the nurse take? A. Tell the client that he's probably weak from inactivity. B. Look at the chart for his last potassium level and contact the physician. C. Look at the chart for his last digoxin level and notify the physician. D. Notify the physician that the client is experiencing heart failure.

B. Look at the chart for his last potassium level and contact the physician.

A client who underwent abdominal surgery returns from the postanesthesia care unit with a nasogastric (NG) tube in place. The client complains of nausea. While preparing to attach the client's NG tube to intermittent suction, the nurse notices that the ground on the suction machine's plug is broken. What priority action should the nurse perform first? A. Use the machine as is because the client is nauseous. B. Obtain another machine from central supply. C. Tape the broken ground to the plug and use the machine. D. Report the problem to the supervisor.

B. Obtain another machine from central supply.

While preparing to start a stat I.V. infusion, a nurse notices a broken ground wire on the infusion pump's plug. What would the nurse do first? A. Continue to use the infusion pump. B. Obtain another pump from central supply. C. Pull the pump out of service. D. Report the problem to central supply.

B. Obtain another pump from central supply.

A nurse is administering eye drops to a client. Which technique is correct? A. Have the client look upward and drop medication into the inner canthus. B. Pull the lower lid down, press the tear duct, and drop medication into the conjunctival sac. C. Hold both lids open and drop medication onto the sclera. D. Tilt the head to the side and drop the medication into the outer canthus.

B. Pull the lower lid down, press the tear duct, and drop medication into the conjunctival sac.

When checking a client's medication profile, the nurse notes that the client is receiving a drug that is contraindicated in clients with glaucoma. The nurse knows that this client has a history of glaucoma and has been receiving the medication for the past 3 days. What should the nurse do first? A. Continue to give the medication because the client has been receiving it for 3 days. B. Report the information to the physician to ensure client safety. C. File an incident report because several other staff members gave the medication. D. Find out whether there are extenuating reasons for giving the drug to this client.

B. Report the information to the physician to ensure client safety.

A newly pregnant client tells a nurse that she has not been taking her prenatal vitamins because they make her feel nauseated. What information should the nurse reinforce to the client? A. Switch to another brand of prenatal vitamins. B. Take the vitamin on a full stomach. C. Consume orange juice with it for better absorption. D. Wait until the morning to take the vitamin.

B. Take the vitamin on a full stomach.

A client is scheduled for a computed tomography (CT) of the chest with contrast media. Which finding should the nurse report immediately to the healthcare provider? A. The client is claustrophobic. B. The client is allergic to shellfish. C. The client recently used a bronchodilator metered-dose inhaler. D. The client is not able to remove a wedding band.

B. The client is allergic to shellfish.

The nurse is administering sublingual nitroglycerin to the client. Immediately after administration, the nurse observes the client for which possible sign or symptom? A. Nervousness or paresthesia B. Throbbing headache or dizziness C. Drowsiness or blurred vision D. Tinnitus or diplopia

B. Throbbing headache or dizziness

A client with rheumatoid arthritis is being discharged with a prescription for aspirin, 600 mg by mouth every 6 hours. The nurse should instruct the client to notify the physician if which adverse drug reaction occurs? A. Dysuria B. Tinnitus C. Leg cramps D. Constipation

B. Tinnitus

To treat a client with acne vulgaris, the physician is most likely to prescribe which topical agent for nightly application? A. Minoxidil B. Tretinoin (retinoic acid) C. Zinc oxide gelatin D. Fluorouracil (5-fluorouracil)

B. Tretinoin (retinoic acid)

A child with iron-deficiency anemia has been prescribed iron supplements. The child's parent brings the child to the clinic 3 months later, and the child's hematocrit remains about the same. What information should the nurse first elicit from the parent? A. a dietary history B. a description of the child's stools C. a history of exposure to sickle cell disease D. whether the child is pale and more tired than usual

B. a description of the child's stools

One hour after receiving pyridostigmine, a client reports difficulty swallowing and excessive respiratory secretions. The nurse notifies the health care provider and prepares to administer which medication? A. additional pyridostigmine B. atropine C. edrophonium D. acyclovir

B. atropine

For a hospitalized client, the health care provider (HCP) orders morphine, 4 mg IM, every 4 hours as needed for pain. However, the client refuses to take injections. Which nursing action is most appropriate? A. administering the medication as ordered B. calling the HCP to request an oral pain medication C. withholding the medication until the client understands its importance D. explaining that no other medication can be given until the client receives the pain medication

B. calling the HCP to request an oral pain medication

The licensed practical nurse (LPN) is coassigned with a registered nurse (RN) who is administering I.V. hydrocortisone; 40 mg b.i.d. Which laboratory value would the nurses expect to be elevated as a result of this medication regimen? A. calcium B. glucose C. magnesium D. potassium

B. glucose

The nurse is caring for a client diagnosed with leukemia who is going to have a chemotherapy treatment. Which test would the nurse expect to be done to evaluate the client's ability to metabolize chemotherapeutic agents? A. lumbar puncture B. liver function studies C. complete blood count (CBC) D. peripheral blood smear

B. liver function studies

The nurse makes initial rounds for the clients. Five medications are scheduled for administration at the same time to five different clients. Which medication should the nurse administer first after initial rounds? A. a maintenance dose of digoxin to the client with congestive heart failure B. morphine sulfate to a client with a myocardial infarction reporting chest pain C. naproxen to the client with rheumatoid arthritis D. ondansetron to a diabetic client reporting nausea

B. morphine sulfate to a client with a myocardial infarction reporting chest pain

An LPN is assisting an RN in discharge teaching for a client who has been prescribed prednisone by the health care provider. The LPN would expect the teaching to include cautioning the client against simultaneous self-administration of what drug in order to lessen the risk for developing peptic ulcers? A. oral antidiabetic agents B. nonsteroidal anti-inflammatory drugs (NSAIDs) C. beta-adrenergic blockers D. oral hormonal contraceptives

B. nonsteroidal anti-inflammatory drugs (NSAIDs)

A 2-year-old child is being treated with rifampin for tuberculosis. Which finding does the nurse expect to find in the client? A. hyperactivity B. orange body secretions C. decreased bilirubin levels D. decreased levels of liver enzymes

B. orange body secretions

A client is receiving captopril for heart failure. Which finding indicates that the medication isn't producing the desired treatment outcome and requires the nurse to notify the health care provider? A. skin rash B. peripheral edema C. dry cough D. orthostatic hypotension

B. peripheral edema

A client diagnosed with bipolar disorder is receiving a maintenance dosage of lithium carbonate. The client is reported by a family member to be hyperactive and hyperverbal. Which intervention is appropriate? A. obtain data regarding mental status B. prepare the client for lithium blood levels C. have the client brought to the local emergency department D. prepare the client for admission to the hospital

B. prepare the client for lithium blood levels

A client in a nursing home is receiving continuous nasogastric (NG) feedings. At the start of the shift, a nurse finds the client turned on the side with the bed flat. The feeding is running with a volumetric pump at 75 mL/hour, as prescribed. The formula container is filled with 150 mL of fluid. Based on this information, which action should the nurse take? A. add 500 mL of formula to the container B. raise the head of the bed (HOB) at least 30 degrees C. turn the client onto the back D. clamp the NG tube

B. raise the head of the bed (HOB) at least 30 degrees

To minimize the amount of a drug received by an infant through breast- feeding, the nurse should tell the mother to: A. take the medication immediately before breast-feeding. B. take the medication immediately after breast-feeding. C. feed the infant 2 hours after taking the medication. D. feed the infant 4 hours after taking the medication.

B. take the medication immediately after breast-feeding.

A nurse inadvertently gives a client a double dose of an ordered medication. After discovering the error, whom should the nurse notify first? A. the client B. the prescriber C. the pharmacist D. the risk manager

B. the prescriber

The primary health care provider prescribes penicillin G, 300,000 units IM, for a child who is 18 months old. Which site does the nurse select to administer this injection? A. deltoid muscle B. vastus lateralis muscle C. dorsogluteal muscle D. ventrogluteal muscle

B. vastus lateralis muscle

The nurse is preparing to calculate the safe dose range of a chemotherapy drug for a child with Wilms tumor. The drug is ordered in mg/m2. What information does the nurse need in order to calculate the body surface area (m2) of the child? Select all that apply. A. blood type B. weight C. height D. white blood cell count (WBC) E. birth weight

B. weight C. height

When reinforcing education with parents of an infant newly diagnosed with diabetes insipidus, which statement by the parent indicates an appropriate understanding of this condition? A. "When my infant stabilizes, I won't have to worry about giving hormone medication." B. "I don't have to measure the amount of fluid intake that I give my infant." C. "I realize that treatment for diabetes insipidus is lifelong." D. "My infant will outgrow this condition."

C. "I realize that treatment for diabetes insipidus is lifelong."

A nurse is reinforcing education for a client about the use of sublingual nitroglycerin. Which statement indicates the client understands the education plan? A. "I must swallow the tablet whole without chewing." B. "I should take a tablet 45 minutes before any strenuous activity." C. "I'll keep the nitroglycerin in its original dark, airtight container." D. "I'll take a tablet every 5 minutes until my chest pain stops."

C. "I'll keep the nitroglycerin in its original dark, airtight container."

For the last 6 days, a 7-month-old infant has been receiving amoxicillin trihydrate to treat an ear infection. Now the parents report redness in the diaper area and small, red patches on the infant's inner thighs and buttocks. After diagnosing Candida albicans, the physician prescribes topical nystatin to be applied to the perineum four times daily. Medication teaching should include which instruction? A. "Discontinue amoxicillin until the diaper area is no longer red." B. "Avoid using superabsorbent disposable diapers." C. "Inspect your infant's mouth for white patches." D. "Switch your infant to a soy-based formula."

C. "Inspect your infant's mouth for white patches."

A client with acquired immunodeficiency syndrome (AIDS) is prescribed zidovudine (azidothymidine, AZT), 200 mg by mouth every 4 hours. When teaching the client about this drug, the nurse should provide which instruction? A. "Take zidovudine with meals." B. "Take zidovudine on an empty stomach." C. "Take zidovudine exactly as prescribed." D. "Take over-the-counter (OTC) drugs to treat minor adverse reactions."

C. "Take zidovudine exactly as prescribed."

A client at 32 weeks' gestation is leaking amniotic fluid and placed on an electronic fetal monitor. The nurse determines that the monitor strip indicates uterine irritability, with contractions occurring every 4 to 6 minutes. The health care provider prescribes magnesium sulfate. When the nurse is reinforcing information about the medication, which statement would be most appropriate? A. "This medicine will help you breathe a whole lot better." B. "You'll probably feel no different than if you had taken acetaminophen." C. "You may feel flushing with a dry mouth." D. "This medicine will make you feel hyperactive and jittery."

C. "You may feel flushing with a dry mouth."

The nurse is precepting a graduate nurse and preparing to give infant immunizations. The preceptor asks the graduate, "Infant injections should only be given in which muscle?" What is the best response by the graduate nurse? A. "rectus femoris" B. "deltoid" C. "vastus lateralis" D. "gluteus maximus"

C. "vastus lateralis"

The physician prescribes 20 units of U-100 regular insulin for a client. The only syringe available is a 1-ml tuberculin syringe. How many milliliters of insulin should the nurse administer? A. 20 B. 2 C. 0.2 D. 0.002

C. 0.2

The label of a drug package reads "hydralazine, 20 mg/ml." How many milliliters would the nurse give a client for a 25-mg dose? A. 0.5 B. 1.0 C. 1.25 D. 1.5

C. 1.25 mL

The physician prescribes an infusion of 2,400 ml of I.V. fluid over 24 hours, with half this amount to be infused over the first 10 hours. During the first 10 hours, the client should receive how many milliliters of I.V. fluid per hour? A. 50 ml/hour B. 100 ml/hour C. 120 ml/hour D. 240 ml/hour

C. 120 ml/hour

The physician orders dextrose 5% in water, 1,000 ml to be infused over 8 hours. The I.V. tubing delivers 15 drops/ml. The nurse should check that the I.V. is infusing at a rate of: A. 15 drops/minute. B. 21 drops/minute. C. 32 drops/minute. D. 125 drops/minute.

C. 32 drops/minute.

A nurse is preparing to administer oral doxycycline to a client. What is the nurse's appropriate action? A. Administer with food. B. Administer with milk. C. Administer with full glass of water. D. Administer with an antacid.

C. Administer with full glass of water.

When reviewing medications for a pharmacology examination, the nursing student recognizes which drugs may be abused because of tolerance and physiologic dependence? A. Lithium and divalproex B. Verapamil and chlorpromazine C. Alprazolam and phenobarbital D. Clozapine and amitriptyline

C. Alprazolam and phenobarbital

The nurse is preparing to discharge a child who has rheumatic fever. Which medication would the nurse expect to be prescribed to prevent recurrence of rheumatic fever? A. Glucocorticoids B. Digoxin C. Antibiotics D. Anti-inflammatory medications

C. Antibiotics

A client with left hemiparesis is having difficulty swallowing a potassium chloride 20 mEq tablet. What should the nurse do? A. Crush the pill and administer with a small amount of liquid. B. Break the pill into small pieces and administer with apple sauce. C. Ask the health care provider for an order to administer a different consistency through a different route. D. Administer the medication with a large amount of liquid.

C. Ask the health care provider for an order to administer a different consistency through a different route.

The nurse gives a client 0.25 mg of digoxin instead of the prescribed dose of 0.125 mg. What action should the nurse take after realizing the mistake? A. Give the prescribed 0.125 mg as soon as possible. B. No action is needed because of the small dose difference. C. Assess the client and notify the physician. D. Hold the next dose of digoxin.

C. Assess the client and notify the physician.

After total hip replacement, a client is receiving epidural analgesia to relieve pain. Which of the following is a nursing priority for this client? A. Changing the catheter site dressing every shift B. Assessing capillary refill time C. Assessing for sensation in the legs D. Keeping the client flat in bed

C. Assessing for sensation in the legs

When checking a client's I.V. insertion site, the nurse notes normal color and temperature at the site and no swelling. However, the I.V. solutions haven't infused at the ordered rate; the flow rate is slow even with the roller clamp wide open. When the nurse lowers the I.V. fluid bag, no blood returns to the tubing. What should the nurse do first? A. Discontinue the I.V. infusion at that site and have it restarted it in the other arm. B. Irrigate the I.V. tubing with 1 ml of normal saline solution. C. Check the tubing for kinks and reposition the client's wrist and elbow. D. Elevate the I.V. fluid bag.

C. Check the tubing for kinks and reposition the client's wrist and elbow.

A client who has been recently diagnosed with gout asks the nurse to explain why they need to take colchicine. What should the nurse base the response on? A. Colchicine increases estrogen levels in the bloodstream. B. Colchicine decreases the risk of infection. C. Colchicine decreases inflammation. D. Colchicine decreases bone demineralization.

C. Colchicine decreases inflammation.

A client who takes over-the-counter drugs regularly is seen at a clinic. The nurse should take which actions to ascertain the client's safety when taking these drugs? Select all that apply. A. Determine whether the drugs are expensive B. Determine whether the drugs are generic C. Determine whether the client knows the appropriate drug dosages and administration schedules D. Determine whether the client knows that these drugs are available in the hospital E. Determine whether the client knows the correct reason for using the drug and its proper route of administration

C. Determine whether the client knows the appropriate drug dosages and administration schedules E. Determine whether the client knows the correct reason for using the drug and its proper route of administration

A nurse is working in a clinic where a family member's spouse is treated for a sexually transmitted disease. The nurse is concerned about the risk to family members. What is the most appropriate action for the nurse to take? A. Anonymously inform the family member of the spouse's diagnosis so that they may seek necessary treatment. B. As legally required, inform the family member of the client's diagnosis. C. Encourage the client to speak with the family member about the diagnosis if the client has not already done so. D. Provide the local Board of Health with the family member's name so they can contact them with information about the client's diagnosis.

C. Encourage the client to speak with the family member about the diagnosis if the client has not already done so.

A client begins clozapine therapy after several other antipsychotic agents fail to relieve her psychotic symptoms. The nurse instructs her to return for weekly white blood cell (WBC) counts to assess for which adverse reaction? A. Hepatitis B. Infection C. Granulocytopenia D. Systemic dermatitis

C. Granulocytopenia

A client with cancer of the stomach tells the nurse, "I cannot bear the pain anymore. Please give me some poison to free myself from this agonizing pain." The nurse faces a value conflict. Which is true in such a condition? A. The nurse should solely consider the values of the client. B. Value conflict has no effect on the client's compliance. C. Human need may affect the values conflict. D. Values conflict is always destructive in nature.

C. Human need may affect the values conflict.

A child with type 1 diabetes develops diabetic ketoacidosis and receives a continuous insulin infusion. Which condition represents the greatest risk to this child? A. Hypercalcemia B. Hyperphosphatemia C. Hypokalemia D. Hypernatremia

C. Hypokalemia

The newly diagnosed diabetes mellitus client is to receive his first insulin dose. The nurse administers the insulin into his abdomen. How does absorption at the abdominal site compare to absorption at other sites? A. Insulin is absorbed more slowly at abdominal injection sites than at other sites. B. Insulin is absorbed rapidly regardless of the injection site. C. Insulin is absorbed more rapidly at abdominal injection sites than at other sites. D. Insulin is absorbed unpredictably at all injection sites.

C. Insulin is absorbed more rapidly at abdominal injection sites than at other sites.

The nurse has just administered a drug to a child. Which organ is most responsible for drug excretion in children? A. Heart B. Lungs C. Kidneys D. Liver

C. Kidneys

The nurse administers furosemide to treat a client with heart failure. Which adverse effect must the nurse watch for most carefully? A. Increase in blood pressure B. Increase in blood volume C. Low serum potassium level D. High serum sodium level

C. Low serum potassium level

A client had gastric bypass surgery, is on nothing-by-mouth (NPO) status, and is in pain. The nurse gives morphine 4 mg as ordered. In 20 minutes, the client reports feeling nauseous. What would the nurse suspect as the most likely cause? A. The surgery is causing nausea. B. Being NPO, the increase in gastric secretions is precipitating this symptom. C. Morphine, which was given for pain, has a tendency to cause nausea. D. It is a reaction to blood still remaining in the mouth after extubation.

C. Morphine, which was given for pain, has a tendency to cause nausea.

A client is scheduled for surgery at 8 a.m. While completing the preoperative checklist, the nurse sees that the surgical consent form hasn't been signed. It's time to administer the preoperative analgesic. Which nursing action takes the highest priority in this situation? A. Giving the preoperative analgesic at the scheduled time B. Asking the client to sign the consent form C. Notifying the surgeon that the consent form hasn't been signed D. Canceling the surgery

C. Notifying the surgeon that the consent form hasn't been signed

A client who is receiving cyclosporine must practice good oral hygiene, including regular brushing and flossing of the teeth, to minimize gingival hyperplasia. Good oral hygiene also is essential to minimize gingival hyperplasia during long-term therapy with certain drugs. Which of the following drugs falls into this category? A. Procainamide B. Azathioprine C. Phenytoin D. Allopurinol

C. Phenytoin

A client has been receiving chemotherapy to treat cancer. Which data collection finding suggests that the client has developed stomatitis (inflammation of the mouth)? A. White, cottage cheese-like patches on the tongue B. Yellow tooth discoloration C. Red, open sores on the oral mucosa D. Rust-colored sputum

C. Red, open sores on the oral mucosa

The nurse is developing a teaching plan for a client who has just been diagnosed with breast cancer. The nurse should expect the health care practitioner to prescribe which medication? A. Acetaminophen B. Dopamine C. Tamoxifen D. Progesterone

C. Tamoxifen

The licensed practical nurse (LPN) is performing a purified protein derivative (PPD) test on a nursing home resident. Which statements about this test are correct? Select all that apply. A. A PPD test is done to test for allergies. B. Always aspirate before injecting the PPD solution. C. The PPD test is an intradermal test. D. Hold the syringe at a 45° angle to the skin. E. The preferred injection site is the ventral surface of the forearm. F. No wheal should appear at the site following injection.

C. The PPD test is an intradermal test. E. The preferred injection site is the ventral surface of the forearm.

A client has been prescribed a new drug for hypertension. Thirty minutes after taking the drug, the client develops chest tightness, becomes short of breath and tachypneic, and exhibits an altered level of consciousness. What is the best explanation for these symptoms? A. The client is having an asthma attack. B. The client is having a pulmonary embolism. C. The client is experiencing a hypersensitivity reaction to the medication. D. The client is suffering from rheumatoid arthritis.

C. The client is experiencing a hypersensitivity reaction to the medication.

Teaching for women in their childbearing years who are receiving antipsychotic medications should include which of the following facts? A. Increased libido is an adverse effect of these medications. B. Incidence of dysmenorrhea increases. C. The client should continue using contraception during periods of amenorrhea. D. Amenorrhea is irreversible.

C. The client should continue using contraception during periods of amenorrhea.

A primary health care provider has instructed a client to check the radial pulse each morning before taking digoxin. After the nurse reinforces education with the client on how to take a radial pulse, which client behavior indicates an accurate understanding of the technique? A. The client places a stethoscope over the apex of the heart. B. The client counts the radial pulse for 10 seconds and multiplies by 6. C. The client uses the middle three fingertips to palpate the radial artery. D. The client states that the radial pulse should be taken immediately after breakfast.

C. The client uses the middle three fingertips to palpate the radial artery.

A medication order reads, "Meperidine 1 ml I.M. stat." A nurse responsible for administering the drug should base the next action on which understanding? A. The order should specify the precise time to give the drug. B. The ordered route is inappropriate for administration of this drug. C. The nurse should clarify the order with the physician. D. The order is correct and valid.

C. The nurse should clarify the order with the physician.

A client with hypothyroidism is prescribed levothyroxine 0.05 mg by mouth daily before breakfast. As the nurse gives the client the medication, the client states, "What dose am I getting? I've been taking 0.15 mg every day for years." Which action by the nurse is most appropriate? A. Tell the client that the primary health care provider must have lowered the dose. B. Administer the medication. C. Verify the dose against the health care provider's prescription in the client's medical record. D. Check the prescription against what is written on the medication administration record.

C. Verify the dose against the health care provider's prescription in the client's medical record.

The nurse is preparing to administer a dose of chlorpropamide to a client with type 2 diabetes. Before administering the drug, the nurse checks the client's allergies and notices that the client is wearing an allergy alert bracelet that indicates an allergy to sulfa drugs. Which action should the nurse take? A. Administer the drug as prescribed. B. Notify the nursing supervisor immediately. C. Withhold the drug and notify the health care provider. D. Confirm that the dosage is correct, then administer the drug.

C. Withhold the drug and notify the health care provider.

notes that the client has chills, dyspnea, and urticaria. The nurse reports this to the physician immediately because the client probably is experiencing: A. a hemolytic reaction to mismatched blood. B. a hemolytic reaction to Rh-incompatible blood. C. a hemolytic allergic reaction caused by an antigen reaction. D. a hemolytic reaction caused by bacterial contamination of donor blood.

C. a hemolytic allergic reaction caused by an antigen reaction.

After intentionally taking an overdose of amitriptyline, a client is admitted to the emergency department. The nurse knows which action will the activated charcoal display when administered to this client? A. cause vomiting of the ingested drug B. stimulate bowel motility for rapid excretion of the drug C. bind with the ingested drug D. neutralize the ingested drug

C. bind with the ingested drug

A pregnant client is receiving heparin. While the client is receiving this drug, which data would the nurse immediately report to the supervising nurse? A. in fetal activity and position B. increase in blood pressure and temperature C. bleeding from an orifice D. change in uterine contraction intensity

C. bleeding from an orifice

While obtaining data from a client, the nurse observes that the client has been prescribed tadalafil. What should the nurse carefully monitor for this client? A. urine output B. temperature C. blood pressure D. urine specific gravity

C. blood pressure

The nurse administers nalbuphine hydrochloride to a postoperative client. Which finding indicates to the nurse that the client is responding as expected to the medication? A. pulse oximetry reading 89% B. pulse rate 104 beats/minute and regular C. blood pressure 115/72 mm Hg D. respiratory rate 12 breaths/minute and irregular

C. blood pressure 115/72 mm Hg

A client has just been diagnosed with early glaucoma. During a teaching session, the nurse should: A. provide instructions on eye patching. B. assess the client's visual acuity. C. demonstrate eyedrop instillation. D. teach about intraocular lens cleaning.

C. demonstrate eyedrop instillation.

After administering an I.M. injection, a nurse should A. recap the needle and discard the needle and syringe in any medical waste container. B. recap the needle and discard the needle and syringe in a puncture-proof container. C. discard the uncapped needle and syringe in a puncture-proof container. D. break the needle using the facility-approved device and discard the needle and syringe in any medical waste container.

C. discard the uncapped needle and syringe in a puncture-proof container.

Which information must be included in a medication order? A. drug class B. possible adverse reactions C. health care provider's signature D. client allergies

C. health care provider's signature

A pregnant client with a history of cardiac dysfunction has been taking propranolol, a beta-adrenergic blocker, to treat hypertension. During labor, the nurse should assess for what adverse effect of this drug? A. uterine hypotonus B. uterine hypertonus C. hypotension D. tachycardia

C. hypotension

The nurse is working in the emergency department when a child is admitted in sickle cell crisis. Which intervention should the nurse expect to perform? A. give blood transfusions B. give antibiotics C. increase fluid intake and give analgesics D. prepare the child for a splenectomy

C. increase fluid intake and give analgesics

A licensed practical nurse is providing care for a client who is undergoing opiate withdrawal. The client is receiving medication therapy to minimize the effects. Which drug would the nurse expect to administer to the client? A. phenobarbital B. dextroamphetamine C. methadone D. diazepam

C. methadone

A client has been diagnosed with tuberculosis (TB). Which pharmacologic therapy does the nurse anticipate administering? A. theophylline B. penicillin intramuscular (IM) C. rifampin, isoniazid, and rifapentine D. Aerosol treatments with pentamidine

C. rifampin, isoniazid, and rifapentine

Which type of medication order might read "Vitamin K 10 mg I.M. daily × 3 days?" A. one-time order B. stat order C. standing order D. as-needed order

C. standing order

The health care provider has ordered sulfasalazine for a child with juvenile rheumatoid arthritis. The nurse questions the order when reading that the client has an allergy to what medication? A. alprazolam B. naproxen C. sulfamethoxazole-trimethoprim D. penicillin

C. sulfamethoxazole-trimethoprim

The physician prescribes 250 mg of a drug. The drug vial reads 500 mg/ml. How much of the drug should the nurse give? A. 2 ml B. 1 ml C. ½ ml D. ¼ ml

C. ½ ml

The physician orders heparin, 7,500 units, to be administered subcutaneously every 6 hours. The vial reads 10,000 units per milliliter. The nurse should anticipate giving how much heparin for each dose? A. ¼ ml B. ½ ml C. ¾ ml D. 1¼ ml

C. ¾ ml

The nurse is teaching a client who will be discharged with a prescription for warfarin. Which statement by the client indicates understanding? A. "I should increase my intake of yogurt and broccoli." B. "This drug will dissolve any clots I may still have." C. "If I miss a dose, I should double the next dose." D. "I should avoid aspirin while taking warfarin."

D. "I should avoid aspirin while taking warfarin."

A nurse is reinforcing education for a client who has been prescribed buspirone for long-term treatment of anxiety. The nurse determines that the education has been effective when which statement is made by the client? A. "I will take the medicine only when I feel an anxiety attack coming on." B. "I will not take the medicine with my meals." C. "I will not stop the medicine if I become pregnant." D. "I will not take the medicine with grapefruit juice."

D. "I will not take the medicine with grapefruit juice."

During the client-teaching session, which instruction should the nurse give to a client receiving alprazolam? A. "Discontinue the medication immediately if you experience nausea." B. "Notify the physician if you experience urine retention." C. "Apply sunscreen to prevent photosensitivity." D. "Inform the physician if you become pregnant or intend to do so."

D. "Inform the physician if you become pregnant or intend to do so."

A nurse is to administer 1,000 ml of normal saline over 6 hours to a client in labor. The drip factor of the IV administration set is 15 drops/ml. What is the rate of the infusion? A. 31 drops/minute B. 35 drops/minute C. 39 drops/minute D. 42 drops/minute

D. 42 drops/minute

When giving an intramuscular (IM) injection, which angle should the nurse insert the needle into the muscle? A. 15 degrees B. 30 degrees C. 45 degrees D. 90 degrees

D. 90 degrees

A child experiences nausea and vomiting after receiving cancer chemotherapy drugs. What should the nurse do to help prevent these problems from recurring? A. Keep the child NPO for 8 hours after the chemotherapy session. B. Administer ordered buspirone 2 hours before the next chemotherapy session. C. Encourage increased fluid intake before the next chemotherapy session. D. Administer ondansetron 30 to 60 minutes before the next chemotherapy session.

D. Administer ondansetron 30 to 60 minutes before the next chemotherapy session.

When drawing up a medication, the nurse notes there are small air bubbles adhering to the interior surface of the syringe. The nurse knows which effect the bubbles might have on parenteral administration? A. Altered onset of action B. Altered duration C. Altered drug absorption D. Altered drug dose

D. Altered drug dose

When administering morphine (the drug of choice for moderate to severe pain in pediatric patients), to a school-age child, which symptom should cause the nurse to be concerned? A. Constipation B. Nausea and vomiting C. Pruritus D. Anemia

D. Anemia

The nurse is teaching a client how to administer subcutaneous (subQ) insulin injections. Which injection site would be appropriate for the client to use? A. Deltoid B. Rectus femoris C. Vastus lateralis D. Anterior aspect of the thigh

D. Anterior aspect of the thigh

While studying for an upcoming examination, which instances would the nursing student review in which sedative-hypnotic drugs are indicated? A. Obsessive-compulsive disorder (OCD) B. Attention deficit hyperactivity disorder (ADHD) C. Hallucinations and delusions D. Anxiety and insomnia

D. Anxiety and insomnia

The nurse monitors a client receiving enoxaparin, 30 mg subQ b.i.d after hip replacement surgery. Which adverse reaction is the client most likely to experience? A. Anaphylactic shock B. Hypersensitivity C. Bronchospasm D. Bleeding

D. Bleeding

The nurse should include which in-home management instruction for a child who's receiving desmopressin acetate for symptomatic control of diabetes insipidus? A. Give desmopressin acetate only when urine output begins to decrease. B. Clean skin with alcohol before applying the desmopressin acetate dermal patch. C. Increase the desmopressin acetate dose if polyuria occurs just before the next scheduled dose. D. Call the health care provider for an alternate route of desmopressin acetate when the child has an upper respiratory infection (URI) or allergic rhinitis.

D. Call the health care provider for an alternate route of desmopressin acetate when the child has an upper respiratory infection (URI) or allergic rhinitis.

A client reports difficulty swallowing when the nurse tries to administer a medication in capsule form. What action should the nurse take to resolve this problem? A. Dissolve the capsule in a full glass of water. B. Break the capsule and give the contents with applesauce. C. Withhold the medication. D. Check for availability of a liquid preparation.

D. Check for availability of a liquid preparation.

A nurse is supervising a student during medication administration to a client. Which action by the student would cause the nurse to intervene during the med pass at the bedside? A. Ask the client's name. B. Check the client's identification band. C. Compare data to the medication administration record. D. Check the room number and the client's name on the bed.

D. Check the room number and the client's name on the bed.

The health care provider prescribes a digoxin elixir for a toddler with heart failure. What action should the nurse take first before administering this drug? A. Check the potassium level. B. Assess blood pressure. C. Obtain a daily weight. D. Determine the apical pulse.

D. Determine the apical pulse.

A client is receiving chemotherapy to treat breast cancer. Which data collection finding indicates a fluid and electrolyte imbalance induced by chemotherapy? A. Urine output of 400 ml in 8 hours B. Serum potassium level of 3.6 mEq/L C. Blood pressure of 120/64 to 130/72 mm Hg D. Dry oral mucous membranes and cracked lips

D. Dry oral mucous membranes and cracked lips

A client receives morphine 4 mg intravenous (I.V.) for relief of surgical pain. Thirty minutes later, as part of which step of the nursing process, the nurse asks the client to rate his level of pain using a pain scale? A. Data collection B. Planning C. Implementation D. Evaluation

D. Evaluation

A mother brings her preschool child to the emergency department after he ingested an unknown quantity of acetaminophen. Which treatment will the physician probably prescribe next? A. Administration of a dose of ipecac syrup B. Insertion of a nasogastric tube and administration of an antacid C. I.V. infusion of normal saline solution D. Gastric lavage and administration of acetylcysteine

D. Gastric lavage and administration of acetylcysteine

The nurse is reviewing laboratory values on a client with heart failure. The client has a potassium level of 4.6 mEq/L (4.6 mmol/L). The client is scheduled to receive a 0900 dose of furosemide. What should the nurse do next? A. Withhold the furosemide dose. B. Administer half the furosemide dose. C. Notify the health care provider. D. Give the furosemide dose to the client.

D. Give the furosemide dose to the client.

A client with a history of atrial arrhythmia is receiving propranolol, 10 mg by mouth three times per day. The nurse knows that propranolol inhibits the action of sympathomimetics at beta1-receptor sites. Where are these sites mainly located? A. Uterus B. Blood vessels C. Bronchi D. Heart

D. Heart

A nurse is administering morning medications to a client on warfarin. Upon reviewing the laboratory results, the nurse notes a prothrombin time (PT) of 27.3. What should the nurse do? A. Give warfarin as prescribed. B. Withhold the morning dose of warfarin and give it later in the day. C. Repeat the laboratory result. D. Hold the medication and notify the health care provider.

D. Hold the medication and notify the health care provider.

While out of bed walking, a client reports dizziness and requests to go back to the room. The nurse obtains the blood pressure machine and obtains vital signs on the client. The client's pulse is 50 and the blood pressure machine reads 80/40 mmHg. The nurse notes the client is scheduled to receive verapamil and atenolol. Which actions by the nurse are best? Select all that apply. A. Give the medications and check vital signs later. B. Call the supervisor and ask what to do. C. Give the scheduled medications. D. Hold the medications. E. Call the healthcare provider and provide a report of the events and vital signs.

D. Hold the medications. E. Call the healthcare provider and provide a report of the events and vital signs.

A client is receiving a blood transfusion. If this client experiences an acute hemolytic reaction, which nursing intervention is the most important? A. Immediately stop the transfusion, infuse dextrose 5% in water (D5W), and call the physician. B. Slow the transfusion and monitor the client closely. C. Stop the transfusion, notify the blood bank, and administer antihistamines. D. Immediately stop the transfusion, infuse normal saline solution, notify the blood bank, and call the physician.

D. Immediately stop the transfusion, infuse normal saline solution, notify the blood bank, and call the physician.

A client reports a severe headache and blurred vision. The nurse immediately obtains vital signs, which reveals a blood pressure of 192/110 mm Hg. The nurse reviews the client's medical record and notes a prescription for clonidine 0.1 mg by mouth as needed for systolic blood pressure greater than 170 mm Hg and diastolic blood pressure greater than 100 mm Hg. The nurse checks the client's medication supply, but no clonidine is available. How should the nurse proceed? A. Notify the primary health care provider, and document that the medication is unavailable. B. Notify the pharmacy, and document in the medical record that the medication is unavailable. C. Call the pharmacy, and ask them to dispense the medication with the next scheduled delivery. D. Inform the pharmacy that the medication is unavailable, ask them to prepare it, and tell them that someone will pick it up immediately.

D. Inform the pharmacy that the medication is unavailable, ask them to prepare it, and tell them that someone will pick it up immediately.

A client who's taking antipsychotic medication develops a very high temperature, severe muscle rigidity, tachycardia, and rapid deterioration in mental status. The nurse suspects what severe complication of antipsychotic therapy? A. Agranulocytosis B. Thrombocytopenia C. Anticholinergic effects D. Neuroleptic malignant syndrome (NMS)

D. Neuroleptic malignant syndrome (NMS)

A nurse administers the client's prescribed antibiotic. The client tells the nurse, "I usually take a white tablet, not a yellow tablet." What is the priority action by the nurse? A. Tell the client that the yellow tablet is from a different manufacturer. B. Reassure the client that the tablet is the correct medication. C. Withhold the medication and notify the health care provider. D. Perform a recheck of the medication name and strength.

D. Perform a recheck of the medication name and strength.

When preparing to administer a drug dose to a client, the nurse examines the drug label. The nurse understands which information that the Food, Drug, and Cosmetic Act (Canada's Food and Drug Act and Regulations) requires drug labels to display? A. Cost per dose B. Date the drug was approved for use by the Food and Drug Administration C. Drug's chemical name D. Presence, quantities, and proportions of certain ingredients

D. Presence, quantities, and proportions of certain ingredients

The nurse is administering theophylline to a client. What is the drug's therapeutic action when used to treat asthma? A. Depresses the myocardium B. Decreases diuresis C. Improves respiratory drive D. Relaxes bronchial smooth muscle

D. Relaxes bronchial smooth muscle

The nurse has an order to administer an intramuscular (I.M.) injection using the Z-track technique. When carrying out this order, what nursing intervention should the nurse implement? A. Insert the needle at a 45-degree angle. B. Wipe the needle immediately after injection. C. Pull the skin laterally toward the injection site. D. Simultaneously withdraw the needle and release the skin.

D. Simultaneously withdraw the needle and release the skin.

When reviewing a client's file, the nurse reviews the following medication order "Vitamin K 10 mg intramuscular (I.M.) daily × 3 days?" The nurse recognizes this as which type of order? A. Single order B. Stat order C. Standing order D. Standard written order

D. Standard written order

A nurse notices the smell of marijuana on a nursing colleague upon return from lunch break. The colleague is having difficulty drawing up a dose of insulin, appears uncoordinated, and is unaware that the needle has been contaminated. What is the best action for the nurse to take? A. Take the syringe and insulin vials, draw up the insulin, and instruct the colleague to focus more clearly when giving the injection. B. Take the insulin vials and needle, draw up the insulin, and administer it. Ask a colleague to observe the nurse for the remainder of the shift. C. Reassign the responsibilities, and inform the colleague that the unit manager will be notified if it occurs again. D. Stop the colleague from drawing up the insulin. Notify the supervisor about the incident, and document the observations.

D. Stop the colleague from drawing up the insulin. Notify the supervisor about the incident, and document the observations.

The nurse is reinforcing education for a client taking tetracycline for severe inflammatory acne. Which instructions are important to reinforce? A. Take the drug with or without meals. B. Take the drug with milk and milk products. C. Take the drug on an empty stomach with small amounts of water. D. Take the drug 1 hour before or 2 hours after meals with large amounts of water.

D. Take the drug 1 hour before or 2 hours after meals with large amounts of water.

A client with persistent, severe schizophrenia has been treated with phenothiazines for the past 17 years. Now the client's speech is garbled as a result of drug-induced rhythmic tongue protrusion. What is another name for this extrapyramidal symptom? A. Dystonia B. Akathisia C. Pseudoparkinsonism D. Tardive dyskinesia

D. Tardive dyskinesia

The nurse is teaching a client how to rotate insulin injection sites. What is the purpose of rotating injection sites? A. To prevent bruising B. To prevent medication leakage from the tissue or muscle C. To prevent erratic drug distribution D. To prevent the formation of hard nodules

D. To prevent the formation of hard nodules

Which information should the nurse include when reinforcing instructions for a client about using vaginal medications? A. Use a tampon after insertion to increase medication absorption. B. Release and pull up on the applicator before removal. C. The suppositories should be kept at room temperature. D. Use a water-soluble lubricant when inserting a suppository.

D. Use a water-soluble lubricant when inserting a suppository.

The nurse has an order to administer an iron dextran 50 mg intramuscular (I.M.) injection. When carrying out this order, which method should the nurse utilize? A. Insert the needle at a 45-degree angle. B. Wipe the needle immediately after injection. C. Pull the skin laterally toward the injection site. D. Use the Z-track technique.

D. Use the Z-track technique.

A nurse is assisting with developing an education plan for a client diagnosed with type 1 diabetes. Which method is most effective for educating the client about self-administration of insulin? A. a short videotape that provides useful information and demonstrations B. an audiotape version of discharge instructions C. a list of instructions written at a sixth-grade reading level D. a discussion and demonstration between the nurse and the client

D. a discussion and demonstration between the nurse and the client

A dopamine receptor agonist such as bromocriptine relieves muscle rigidity caused by antipsychotic medication by: A. blocking dopamine receptors in the central nervous system (CNS). B. blocking acetylcholine in the CNS. C. activating norepinephrine in the CNS. D. activating dopamine receptors in the CNS.

D. activating dopamine receptors in the CNS.

The nurse is providing education to a client who has received a new prescription for oral contraceptives. What would the nurse instruct the client to report to her primary health care provider related to the medication? A. breast tenderness B. cramps during menstruation C. decreased menstrual flow D. blurred vision and severe headache

D. blurred vision and severe headache

The nurse is obtaining data from a group of clients with depression. Which clients would the nurse recognize would most benefit from electroconvulsive therapy (ECT)? A. clients who are suicidal or homicidal B. clients who are aggressive or acting out and depressed C. clients with physical problems resulting in depression D. clients who are severely depressed and do not respond to medication trials

D. clients who are severely depressed and do not respond to medication trials

The client was admitted to the hospital with the diagnosis of iron overload. Over time, an excess of iron can damage the liver and cause heart problems. Which medication does the nurse anticipate the healthcare provider to order? A. montelukast B. ramipril C. flurazepam D. deferoxamine

D. deferoxamine

The nurse is caring for a client who is receiving antibiotics to treat a gram-negative bacterial infection. Because antibiotics destroy the body's normal flora, the nurse must monitor the client for: A. platelet dysfunction. B. oliguria and dysuria. C. stomatitis. D. diarrhea.

D. diarrhea.

When a nurse tries to administer medication, the client refuses it, saying, "I don't have to take those pills if I don't want to." What intervention by the nurse would have the highest priority? A. insisting that the client take the medication because it is specifically ordered for the client B. reporting the client's comments to the physician and the treatment team C. explaining the consequences of not taking the medication, such as a negative outcome D. exploring how the client's feelings affect the decision to refuse medication

D. exploring how the client's feelings affect the decision to refuse medication

A nurse is administering captopril to a client with heart failure. Which evaluation finding should prompt the nurse to withhold the next dose and notify the healthcare provider? A. hypertension B. third heart sound (S3) C. dyspnea D. hyperkalemia

D. hyperkalemia

The nurse is caring for a client who developed ketoacidosis. Which prescribed treatment does the nurse anticipate administering? A. glucagon B. blood products C. glucocorticoids D. insulin and IV fluids

D. insulin and IV fluids

A medication nurse is preparing to administer 9 a.m. medications to a client with liver cancer. Which consideration is the nurse's highest priority? A. frequency of the medication B. purpose of the medication C. necessity of the medication D. metabolism of the medication

D. metabolism of the medication

After experiencing a transient ischemic attack (TIA), a client is prescribed aspirin, 325 mg by mouth daily. The nurse should teach the client that this medication has been prescribed to: A. control headache pain. B. enhance the immune response. C. prevent intracranial bleeding. D. reduce platelet agglutination.

D. reduce platelet agglutination.

A 59-year-old client is scheduled for cardiac catheterization the next morning. His physician prescribed secobarbital sodium, 100 mg by mouth at bedtime, for sedation. Before administering the drug, the nurse should know that: A. sedatives cause predictable responses; hypnotics cause unpredictable ones. B. sedatives interact with few drugs; hypnotics interact with many. C. sedatives don't depress respirations; hypnotics do. D. sedatives reduce excitement; hypnotics induce sleep.

D. sedatives reduce excitement; hypnotics induce sleep.


संबंधित स्टडी सेट्स

Chapter 59 Assessment and Management of Problems Related to Male Reproductive Processes

View Set

Chapter 2 - Network Infrastructure and Documentation

View Set

APUSH Periods 1-8 Identifies by Donald Chau

View Set

Earth Systems & Environment Chapter 11 quiz

View Set

Section 5.5 Part 1: Finding the Mean, Median, and Mode of a Set of Numbers

View Set

CD1 - Ch 12: Intellectual Development from One to Three

View Set